Diễn Đàn MathScopeDiễn Đàn MathScope
  Diễn Đàn MathScope
Ghi Danh Hỏi/Ðáp Community Lịch

Go Back   Diễn Đàn MathScope > Sơ Cấp > Tài Liệu > Đề Thi > Đề Thi HSG Cấp Tỉnh ở Việt Nam

News & Announcements

Ngoài một số quy định đã được nêu trong phần Quy định của Ghi Danh , mọi người tranh thủ bỏ ra 5 phút để đọc thêm một số Quy định sau để khỏi bị treo nick ở MathScope nhé !

* Nội quy MathScope.Org

* Một số quy định chung !

* Quy định về việc viết bài trong diễn đàn MathScope

* Nếu bạn muốn gia nhập đội ngũ BQT thì vui lòng tham gia tại đây

* Những câu hỏi thường gặp

* Về việc viết bài trong Box Đại học và Sau đại học


Trả lời Gởi Ðề Tài Mới
 
Ðiều Chỉnh Xếp Bài
Old 04-10-2014, 07:06 PM   #1
mathandyou
Moderator
 
Tham gia ngày: Dec 2012
Đến từ: HCMUS
Bài gởi: 557
Thanks: 259
Thanked 402 Times in 216 Posts
Sáng nay mới học thầy Dũng,bài PTNK giải khá ảo,xin trình bày lại
Đặt $x=\frac{1}{a},y=\frac{1}{b},z=\frac{1}{c}$ ta có $x+y+z+xy+yz+zx=3$
Ta cần chứng minh $$xy+yz+zx\le xyz+1$$
Từ điều kiện:$x+y+z+xy+yz+zx=3$ thì tồn tại nhiều nhất trog $3$ số $x,y,z$ một số lớn hơn $1$.
Ta xét các khả năng sau:
1)Nếu $x,y,z \leq 1$.
Vì $(x+yz)+(y+zx)+(z+xy)=3$ nên không mất tính tổng quát ta có thể giả sử:$x+yz=min$,suy ra: $1 \geq x+yz$.(1)
Mặt khác thì:$x(y-1)(z-1) \geq 0 \leftrightarrow xyz+x \geq xy+yz$ (2)
Từ (1) và (2) suy ra đpcm.
2)$x>1$,$y,z <1$.
Khi đó: $(x-1)(y-1)(z-1) \geq 0$
$\Leftrightarrow xyz+x+y+z-1 \geq xy+yz+zx$
+)Nếu $x+y+z \leq 2$ thì $xyz+1 \geq xyz+x+y+z-1 \geq xy+yz+zx$
+)Nếu $x+y+z >2$ thì từ $x+y+z+xy+yz+zx=3$ nên $x+y+z <1$ nên ta có đpcm.
[RIGHT][I][B]Nguồn: MathScope.ORG[/B][/I][/RIGHT]
 
__________________
Xét cho cùng, phần thưởng cao quý nhất mà công việc mang lại không phải là thứ bạn nhận được, mà nó vẽ nên chân dung con người bạn ra sao.

[Only registered and activated users can see links. ]
mathandyou is offline   Trả Lời Với Trích Dẫn
The Following 6 Users Say Thank You to mathandyou For This Useful Post:
giabao185 (10-10-2014), Juliel (04-10-2014), Livetolove2207 (10-12-2014), namdung (04-10-2014), osp (05-10-2014), Samurott (05-10-2014)
Old 04-10-2014, 09:10 PM   #2
einstein1996
Senior Member
 
Tham gia ngày: Nov 2011
Đến từ: việt nam
Bài gởi: 103
Thanks: 77
Thanked 43 Times in 28 Posts
Trích:
Nguyên văn bởi namdung View Post
6. (PTNK) Tìm tất cả các hàm số$ f: N^* -> N^*$ thỏa mãn hệ thức $ f(f(n)/n) = n^2 $ với mọi n nguyên dương. N* ký hiệu tập hợp các số nguyên dương.
Làm thử bài 6 như sau:
Đặt $f(n)=ng(n)$ thì $g: N^*-> N^*$.
Khi đó $f(\frac{f(n)}{n})=n^2 \Leftrightarrow g(n)g(g(n))=n^2$.
Lấy logarit hai vế ta có: $ ln{g(n)}+ln{g(g(n))}=2ln{n}$.
Đến đây ta xét dãy sau:
$u_0=ln{x};x\in N^*$ và $u_n=ln{g_n(x)}$ trong đó $g_n(x)=g(...g(n)...)$ với n lần lấy hàm g.
Ta có: $u_n\geqslant0$ và $u_{n+2}+u_{n+1}-2u_n=0$.
Công thức tổng quát: $u_n=\frac{2ln{x}+ln{g(x)}}{3}+\frac{ln{x}-ln{g(x)}}{3}(-2)^n$.
Nếu tồn tại $x$ sao cho $ln{x}-ln{g(x)}<0$ thì $u_{2n}<0$ với n đủ lớn (mt)
Nếu tồn tại $x$ sao cho $ln{x}-ln{g(x)}>0$ thì $u_{2n+1}<0$ với n đủ lớn (mt).
Vậy $ln{g(x)}=ln{x}$ với mọi $x \in N^*$. Suy ra $f(n)=n^2$
[RIGHT][I][B]Nguồn: MathScope.ORG[/B][/I][/RIGHT]
 
einstein1996 is offline   Trả Lời Với Trích Dẫn
The Following 4 Users Say Thank You to einstein1996 For This Useful Post:
falcaono1 (19-10-2014), giabao185 (10-10-2014), luuvanthai (09-10-2014), nqt (05-10-2014)
Old 05-10-2014, 08:28 AM   #3
yeahboy27
+Thành Viên+
 
Tham gia ngày: Sep 2012
Bài gởi: 7
Thanks: 0
Thanked 5 Times in 2 Posts
(Chuyên Hùng Vương - Phú Thọ): Trong một bảng ô vuông kích thước 999×999, mỗi ô được tô bởi một trong 2 màu trắng hoặc đỏ. Gọi T là số bộ (C1,C2,C3) các ô mà hai ô đầu trong cùng 1 hàng và hai ô cuối cùng 1 cột, với C1 và C2 màu trắng, C3 màu đỏ.
Tìm giá trị lớn nhất của T
[RIGHT][I][B]Nguồn: MathScope.ORG[/B][/I][/RIGHT]
 
yeahboy27 is offline   Trả Lời Với Trích Dẫn
The Following 2 Users Say Thank You to yeahboy27 For This Useful Post:
einstein1996 (05-10-2014), thaygiaocht (05-10-2014)
Old 05-10-2014, 01:50 PM   #4
mathandyou
Moderator
 
Tham gia ngày: Dec 2012
Đến từ: HCMUS
Bài gởi: 557
Thanks: 259
Thanked 402 Times in 216 Posts
Em xin điểm qua một đường tròn mà tần số xuất hiện của nó khá nhiều ở các bài đội tuyển lần này.
Đó là đường tròn đường kính $AM$ với $M$ là trung điểm $BC$ trong $\triangle ABC$.Tất nhiên đường tròn này có khá nhiều bài toán xung quanh nhưng trong chủ đề topic em xin nói đến các bài đội tuyển.
Đầu tiên là bài chọn ĐT Ninh Bình

Bài 1-Cho tam giác $ABC$ nhọn,trực tâm $H$.Gọi $D$ là hình chiếu của $A$ trên $BC$,$M$ là trung điểm của $BC$.Giả sử $E,F$ lần lượt là giao điểm của đường tròn ngoại tiếp tam giác $ABC$ với các tia $MH,ED$.Chứng minh: tứ giác $ABFC$ là tứ giác điều hòa.

Mấu chốt của bài này chính là nhận ra $E$ là giao điểm của đường tròn đường kính $AM$ với $(ABC)$ hay $\angle AEF=90$.Đây cũng là một tính chất rất thú vị.Mọi người có thể xem ở [Only registered and activated users can see links. ].



Cũng trong đề của Ninh Bình tiếp tục xuất hiện đường tròn đường kính $AM$.

Bài 2-Cho tam giác $ABC$,$D$ là trung điểm của cạnh $BC$,$E$ và $F$ lần lượt là hình chiếu của $D$ trên $AB$ và $AC$.Gọi $T$ là giao điểm của các tiếp tuyến tại $E,F$ của đường tròn đường kính $AD$.Chứng minh:$TB=TC$.

Thực ra đây là $1$ tính chất của đường tròn đường kính $AM$ này.



Đường thẳng qua $A$ song song $BC$ cắt đường tròn đường kính $AM$ tại $P$.
Vì $BC \parallel AP$ và $D$ là trung điểm $BC$ nên theo tính chất của chùm điều hòa thì: $(AB,AC,AD,AE)=-1$ hay $(AF,AE,AD,AP)=-1$
Suy ra tứ giác $FDEP$ là tứ giác điều hòa.
Từ đó theo tính chất tứ giác điều hòa thì $T,D,P$ thẳng hàng.(1)
Mặt khác: $PD \perp AP$ nên $PD \perp BC$ hay $PD$ là đường trung trực $BC$.(2)
Do đó từ (1) và (2) ta có ngay $TB=TC$

Với cách làm tương tự bài Ninh Bình thì ta đến với bài hình câu a) của Vũng Tàu

Bài 3-Cho $\triangle ABC$,gọi $D$ là hình chiếu của $A$ lên $BC$ và $M$ là trung điểm $BC$.$E,F$ lần lượt là hình chiếu của $M$ lên $AB,AC$.Đường thẳng vuông góc với $AM$ tại $M$ cắt $EF$ tại $K$.Chứng minh $KA=KD$.



Kẻ đường thẳng qua $A$ song song với $BC$ cắt đường tròn đường kính $AM$ ở $P$.Ta có:$EMFP$ là tứ giác điều hòa.
Nên $K$ sẽ thuộc vào tiếp tuyến tại $P$ của đường tròn đường kính $AM$.Từ đây suy ra $KP=KM$
Mà $APMD$ là hình chữ nhật suy ra $KA=KD$

Hoặc nếu chú ý một chút,ta sẽ xuất phát từ bài Ninh Bình.
Gọi $K'$ là giao của 2 tiếp tuyến tại $P$ và $D$ của $(AM)$.
Vì $T \in PD$ là đường đối cực của $K'$ nên $K'$ thuộc đường đối cực của $T$ hay $K' \in EF$. Hay $K \equiv K'$ nên có đpcm.

Bài này một bạn đã có mở rộng như sau:
Cho tam giác $ABC$,trung điểm $M$ của $BC$,trực tâm $H$ của tam giác $ABC$.Trên $MH$ lấy điểm $P$ .Đường tròn đường kính $AP$ cắt $AB;AC;AH$ lần lượt ở $E;F;D$.Trên cung $EF$ không chứa $A$ lấy điểm $Q$ sao cho $QM$ vuông góc $BC$,tiếp tuyến tại $Q$ của $(AP)$ cắt $EF$ ở $K$,khi đó $KD=KA$.

Và cuối cùng là bài của PTNK cũng có bóng dáng của đường tròn này,dù nó đóng góp không quá nhiều.

Bài 4-Cho $\triangle ABC$ không cân.Gọi $I$ là trung điểm $BC$.Đường tròn $(I,IA)$ cắt $AB,AC$ lần lượt tại $M,N$.$MI,NI$ lần lượt cắt $(I)$ tại $P,Q$.Gọi $K$ là giao của $PQ$ với tiếp tuyến tại $A$ của $(I)$.Chứng minh $K$ nằm trên $BC$.

Ta dễ thấy ngay $KA$ là trục đẳng phương của $(I)$ và đường tròn đường kính $AI$.Từ đó ý tưởng phương tích-trục đẳng phương là sáng sủa nhất.



Kẻ đường kính $AA'$ của $(I)$.Khi đó các tứ giác:$AMA'P$,$ANA'Q$ và $ABA'C$ là hình bình hành nên $A',C,P$ và $A',B,Q$ thẳng hàng.
Từ đó $\angle AQB=\angle APC=90$.
Kẻ đường cao $AH$ thì $H \in (ABQ),(ACP)$
Từ đó: $\angle QHB=\angle QAB=\angle QPM$
Suy ra $QHIP$ là tứ giác nội tiếp.
Do đó $HI$ là trục đẳng phương của đường tròn đường kính $AM$ và $(QHIP)$.
Và thêm nữa là $PQ$ là trục đẳng phương của $(I)$ và $(QHIP)$.
Từ đó thì $AK,PQ,HI$ đồng qui nên ta có đpcm.

P/s:Hồi đó nhớ là Vũng Tàu cũng có $1$ bài chọn đội tuyển cũng liên quan đến đường tròn này.Bài đó thế này,đây cũng là 1 bài toán thú vị

Bài toán
Cho $H$ là trực tâm của tam giác $ABC$ không cân và $\angle A$ nhọn, hình chiếu của $H$ trên $AB,AC$ theo thứ tự là $E,F$. Gọi $D$ là trung điểm của đoạn thẳng $BC$; $P,Q$ là giao điểm của các đường tròn đường kính $AD$, đường kính $BC$.
Chứng minh $H, P, Q$ thẳng hàng và các đường thẳng $BC,EF,PQ$ đồng qui.
[RIGHT][I][B]Nguồn: MathScope.ORG[/B][/I][/RIGHT]
 
Hình Kèm Theo
Kiểu File : jpg ninhbinh2.jpg (29.1 KB, 895 lần tải)
Kiểu File : jpg vungtau.jpg (24.4 KB, 896 lần tải)
Kiểu File : jpg PTNK.jpg (57.4 KB, 897 lần tải)
__________________
Xét cho cùng, phần thưởng cao quý nhất mà công việc mang lại không phải là thứ bạn nhận được, mà nó vẽ nên chân dung con người bạn ra sao.

[Only registered and activated users can see links. ]

thay đổi nội dung bởi: mathandyou, 05-10-2014 lúc 05:41 PM
mathandyou is offline   Trả Lời Với Trích Dẫn
The Following 13 Users Say Thank You to mathandyou For This Useful Post:
9nho10mong (14-10-2014), babysama (10-02-2015), CTK9 (19-10-2014), davidsilva98 (05-10-2014), HoangHungChels (06-10-2014), namdung (05-10-2014), Samurott (05-10-2014), Saruka 01 (24-10-2014), sieusieu90 (02-07-2015), Thmcuongvn (19-10-2014), thuynv (28-10-2014), tson1997 (05-10-2014), tuananhst2000 (28-05-2015)
Old 05-10-2014, 06:01 PM   #5
quocbaoct10
+Thành Viên Danh Dự+
 
quocbaoct10's Avatar
 
Tham gia ngày: Oct 2012
Đến từ: THPT chuyên Lê Quý Đôn-Nha Trang-Khánh Hòa
Bài gởi: 539
Thanks: 292
Thanked 365 Times in 217 Posts
Số học:
Cho $n\in \mathbb{Z^+}$. CMR: Tồn tại $m\in \mathbb{Z^+}$ thỏa mãn: $2^m \equiv 2015 \pmod{3^n}$ và $2^m\equiv 3^{2015} \pmod{5^n}$
(đề thi chọn đt KHTN)

[RIGHT][I][B]Nguồn: MathScope.ORG[/B][/I][/RIGHT]
 
__________________
i'll try my best.
quocbaoct10 is offline   Trả Lời Với Trích Dẫn
The Following User Says Thank You to quocbaoct10 For This Useful Post:
namdung (14-10-2014)
Old 14-10-2014, 03:52 PM   #6
Fool's theorem
+Thành Viên Danh Dự+
 
Fool's theorem's Avatar
 
Tham gia ngày: Oct 2012
Đến từ: T1 K46 Chuyên ĐHSP Hà Nội
Bài gởi: 187
Thanks: 42
Thanked 192 Times in 101 Posts
Gửi tin nhắn qua Yahoo chát tới Fool's theorem
Trích:
Nguyên văn bởi quocbaoct10 View Post
Số học:
Cho $n\in \mathbb{Z^+}$. CMR: Tồn tại $m\in \mathbb{Z^+}$ thỏa mãn: $2^m \equiv 2015 \pmod{3^n}$ và $2^m\equiv 3^{2015} \pmod{5^n}$
(đề thi chọn đt KHTN)
Mấu chốt bài này là việc $2$ là căn nguyên thủy mod $3^n$ và mod $5^n$ với mọi $n$ nguyên dương (Một số $g$ được gọi là căn nguyên thủy mod $p$ nếu cấp của $g$ mod $p$ là $\phi(p)$). Điều này có thể chứng minh được bằng quy nạp.
Sau đó ta sử dụng một tính chất của căn nguyên thủy là : Nếu $g$ là căn nguyên thủy mod $p$ thì $1,g^1,g^2,...,g^{\phi(p)}$ là một hệ thặng dư thu gọn mod $p$. Do $2015$ nằm trong hệ thặng dư thu gọn mod $3^n$ và $3^{2015}$ nằm trong hệ thặng dư thu gọn mod $5^n$ nên tồn tại $x,y$ sao cho $2^x \equiv 2015 \pmod{3^n}$ và $2^y \equiv 3^{2015} \pmod{5^n}$
Giờ chỉ cần chứng minh tồn tại $m$ nguyên dương sao cho $m \equiv x \pmod{\phi(3^n)}$ và $m \equiv y \pmod{\phi(5^n)}$ là bài toán được giải quyết.
Để có điều này thì ta sẽ áp dụng định lý Thặng dư Trung Hoa tổng quát : Hệ $x \equiv a_i \pmod{b_i}$ sẽ có nghiệm khi và chỉ khi $gcd(b_i,b_j) | a_i-a_j$ với mọi $i,j$ phân biệt.
Thật vậy,
$2^x \equiv 2015 \pmod{3^n} \Rightarrow 2^x \equiv 2 \pmod{3} \Rightarrow x= 2k+1$
$2^y \equiv 3^{2015} \pmod{5^n} \Rightarrow 2^y \equiv 2 \pmod{5} \Rightarrow y=4l+1$
$\Rightarrow gcd(\phi(3^n),\phi(5^n))=2 | x-y \Rightarrow$ hệ $m \equiv x \pmod{\phi(3^n)},m \equiv y \pmod{\phi(5^n)}$ có nghiệm. Ta có đpcm.

Trích:
Nguyên văn bởi yeahboy27 View Post
Hóng mọi người giúp bài này với (Chuyên Hùng Vương - Phú Thọ): Trong một bảng ô vuông kích thước 999×999, mỗi ô được tô bởi một trong 2 màu trắng hoặc đỏ. Gọi T là số bộ (C1,C2,C3) các ô mà hai ô đầu trong cùng 1 hàng và hai ô cuối cùng 1 cột, với C1 và C2 màu trắng, C3 màu đỏ.
Tìm giá trị lớn nhất của T
Mong mọi người gọi ý giúp mình chút.
Đây là bài C3 trong IMO Shortlist 2012
[RIGHT][I][B]Nguồn: MathScope.ORG[/B][/I][/RIGHT]
 
__________________
Hope against hope.
Fool's theorem is offline   Trả Lời Với Trích Dẫn
The Following 2 Users Say Thank You to Fool's theorem For This Useful Post:
quocbaoct10 (19-10-2014), son235 (14-10-2014)
Old 05-10-2014, 09:01 PM   #7
yeahboy27
+Thành Viên+
 
Tham gia ngày: Sep 2012
Bài gởi: 7
Thanks: 0
Thanked 5 Times in 2 Posts
Hóng mọi người giúp bài này với (Chuyên Hùng Vương - Phú Thọ): Trong một bảng ô vuông kích thước 999×999, mỗi ô được tô bởi một trong 2 màu trắng hoặc đỏ. Gọi T là số bộ (C1,C2,C3) các ô mà hai ô đầu trong cùng 1 hàng và hai ô cuối cùng 1 cột, với C1 và C2 màu trắng, C3 màu đỏ.
Tìm giá trị lớn nhất của T
Mong mọi người gọi ý giúp mình chút.
[RIGHT][I][B]Nguồn: MathScope.ORG[/B][/I][/RIGHT]
 
yeahboy27 is offline   Trả Lời Với Trích Dẫn
Old 07-10-2014, 09:33 AM   #8
huynhcongbang
Administrator

 
huynhcongbang's Avatar
 
Tham gia ngày: Feb 2009
Đến từ: Ho Chi Minh City
Bài gởi: 2,413
Thanks: 2,165
Thanked 4,188 Times in 1,381 Posts
Gửi tin nhắn qua Yahoo chát tới huynhcongbang
Đợt này nếu được thì em xin ủng hộ Số học cho đủ bộ ạ. Nhưng chắc là em gom đề và lời giải trên facebook lại rồi gõ thành file thôi chứ giải không hết nổi.
[RIGHT][I][B]Nguồn: MathScope.ORG[/B][/I][/RIGHT]
 
__________________
Sự im lặng của bầy mèo
huynhcongbang is offline   Trả Lời Với Trích Dẫn
The Following 5 Users Say Thank You to huynhcongbang For This Useful Post:
namdung (14-10-2014), quocbaoct10 (07-10-2014), Samurott (07-10-2014), thaygiaocht (07-10-2014), Thmcuongvn (19-10-2014)
Old 14-10-2014, 05:58 AM   #9
namdung
Administrator

 
Tham gia ngày: Feb 2009
Đến từ: Tp Hồ Chí Minh
Bài gởi: 1,343
Thanks: 209
Thanked 4,066 Times in 778 Posts
Gửi tin nhắn qua Yahoo chát tới namdung
Tiếp tục một số bài toán chọn lọc

14. (Daklak) Cho a, b, c > 0. Chứng minh rằng ta có bất đẳng thức
$ \frac{a^2-bc}{\sqrt{8a^2+(b+c)^2}} + \frac{b^2-ca}{\sqrt{8b^2+(c+a)^2}} + \frac{c^2-ab}{\sqrt{8c^2+(a+b)^2}} \ge 0 $

15. (KHTN HN) Cho dãy số $ (x_n) $ xác định bởi: $ x_1 = 2, x_2 = 10, x_{n+2} = \frac{8x^2_{n+1} - x_{n+1}x_n}{x_{n+1}+x_n}, n = 1, 2, 3, ... $. Với mỗi số nguyên dương n, đặt $ y_n = \sum_{k=1}^n \frac{(-1)^k}{x_{k+1}+x_k+3} $. Chứng minh rằng dãy $ y_n $ có giới hạn hữu hạn khi n dần đến vô cùng và tìm giới hạn đó.

16. (ĐH Vinh) Cho a, b, c là các số thuộc đoạn [0, 1]. Tìm giá trị lớn nhất của biểu thức A = (a-b)(b-c)(c-a)(a+b+c).

17. (ĐHSP HN) Cho dãy số $ (x_n) $ xác định bởi $ x_0 = 0 $ và $ x_{n+1} = \frac{7x_n+3\sqrt{4+5x_n^2}}{2} $ với mọi n tự nhiên.
1. Chứng minh rằng với mọi n, $ x_n $ là số tự nhiên và $ x_{2014} $ chia hết cho $ x_{19} $.
2. Chứng minh rằng tồn tại số nguyên dương a sao cho với mọi số nguyên dương n, trong biểu diễn nhị phân của $ x_{an} $ có ít nhất $ 46^{2014} $ chữ số 1.

18. (Thái Bình). Gọi f(n) là số song ánh từ X = {1, 2, ..., n} vào X không có điểm bất động và g(n) là số song ánh từ X vào X có đúng 1 điểm bất động. Chứng minh rằng | f(n) - g(n) | = 1.

19. (ĐHSP HN) Cho dãy số $ (x_n) $ được xác định bởi $ x_1 = 1, x_{n+1} = 5(\sqrt{x_n+11}-\sqrt{x_n+4}) $. Chứng minh rằng dãy $x_n$ có giới hạn hữu hạn và tìm giới hạn đó.

20. (PTNK) Cho X = {1, 2, ..., 19}. Chứng minh rằng tồn tại họ gồm 2600 tập con 7 phần tử của X sao cho với mọi A, B khác nhau thuộc họ này thì A và B có không quá 5 phần tử chung.

21. (PTNK) Tìm số nguyên dương n lớn nhất thỏa mãn các điều kiện sau
i) n không chia hết cho 3;
ii) Bảng vuông n x n ô không thể được phủ kín bằng 1 quân tetramino 1 x 4 và các quân trimino kích thước 1 x 3. Trong phép phủ các quân tetramino và trimino được phép quay dọc nhưng không được phép chườm lên nhau hoặc nằm ra ngoài bảng vuông.

22. (Gia Lai) Xét tất cả các tập hợp $ X = {a_1, a_2, ..., a_n} $ các số nguyên dương có tính chất: khi ta bỏ đi một phần tử bất kỳ trong X thì tập còn lại có thể phân hoạch thành 2 tập con khác rỗng sao cho tổng các phần tử trong mỗi tập con đó bằng nhau. Hãy tìm giá trị nhỏ nhất của n.

23. (Thái Bình) Chứng minh rằng với mỗi số nguyên dương $ n \ge 3 $ phương trình sau $ x^ne^{-x} = 1 $ có nghiệm duy nhất thuộc [0, n]. Tìm $ lim x_n $.

24. Cho số nguyên dương n và các số thực dương $ x_1, x_2, ..., x_n $. Chứng minh rằng
$ (1+x_1)(1+x_1+x_2)...(1+x_1+...+x_n) \ge \sqrt{(n+1)^{n+1}}\sqrt{x_1x_2...x_n} $

25. (ĐHSP HN) Xét đa thức $ P(x) = a_{2n}x^2n + a_{2n-1}x^{2n-1} + ... + a_2x^2 + a_1x + a_0 $ với $ a_i $ thuộc [100, 101]. Tìm n nhỏ nhất sao cho P(x) có nghiệm thực.

26. (KHTN) Cho số nguyên dương n. Chứng minh rằng tồn tại số nguyên dương m sao cho $ 2^ m ≡ 2015 (mod 3^n) $ và $ 2^m ≡ 3^{2015} (mod 5^n) $
[RIGHT][I][B]Nguồn: MathScope.ORG[/B][/I][/RIGHT]
 

thay đổi nội dung bởi: namdung, 14-10-2014 lúc 06:57 AM
namdung is offline   Trả Lời Với Trích Dẫn
The Following 6 Users Say Thank You to namdung For This Useful Post:
9nho10mong (14-10-2014), CTK9 (19-10-2014), DenisO (14-10-2014), luuvanthai (14-10-2014), thaygiaocht (14-10-2014), vantienducdh (26-10-2014)
Old 14-10-2014, 10:33 PM   #10
Short_list
+Thành Viên+
 
Tham gia ngày: May 2012
Đến từ: Tp.HCM
Bài gởi: 85
Thanks: 12
Thanked 79 Times in 32 Posts
Trích:
Nguyên văn bởi namdung View Post
24. Cho số nguyên dương n và các số thực dương $ x_1, x_2, ..., x_n $. Chứng minh rằng
$$ (1+x_1)(1+x_1+x_2)...(1+x_1+...+x_n) \ge \sqrt{(n+1)^{n+1}}\sqrt{x_1x_2...x_n}. \quad (1)$$
Không biết bài này của tỉnh nào ? Ta đặt \[\begin{aligned}
a_1&=\frac{x_1}{1+x_1}\\
a_2&=\frac{x_2}{(1+x_1)(1+x_1+x_2)}\\
&\cdots \\
a_n&=\frac{x_n}{(1+x_1+x_2+\cdots+x_{n-1})(1+x_1+x_2+\cdots + x_n)}\\
a_{n+1}&=\frac{1}{1+x_1+x_2+\cdots+x_n},
\end{aligned}\] khi đó $a_1+a_2+\cdots + a_n+a_{n+1}=1,$ và bất đẳng thức (1) trở thành \[a_1a_2 \cdots a_{n+1} \leqslant \frac{1}{(n+1)^{n+1}}, \quad (2)\] thế nhưng (2) đúng theo bất đẳng thức AM-GM, nên ta có điều phải chứng minh.

P/S: Đây là đề thi của Nga năm 2007, ngoài ra còn một số chứng minh khác và bất đẳng thức vẫn đúng với điều kiện không âm.
------------------------------
Trích:
Nguyên văn bởi namdung View Post
16. (ĐH Vinh) Cho a, b, c là các số thuộc đoạn [0, 1]. Tìm giá trị lớn nhất của biểu thức \[A = (a-b)(b-c)(c-a)(a+b+c).\]
Giả sử $a=\max (a,\, b,\, c).$ Ta chỉ cần xét trường hợp $a \ge c \ge b.$ Khi đó
\[\begin{aligned}
A\leqslant \frac{2(c-b)\cdot (1+\sqrt{3}) (a-c) \cdot (-1+\sqrt{3})(a+b+c)}{4}
\end{aligned}\]
Rồi áp dụng bất đẳng thức AM-GM ta có $P_{\max}=\frac{2\sqrt{3}}{9}$ đạt được khi $(a,\,b,\,c)=(1,\,0,\,\frac{1}{\sqrt{3}}).$
[RIGHT][I][B]Nguồn: MathScope.ORG[/B][/I][/RIGHT]
 
__________________
The Simplest Solution Is The Best Solution

thay đổi nội dung bởi: Short_list, 14-10-2014 lúc 10:46 PM Lý do: Tự động gộp bài
Short_list is offline   Trả Lời Với Trích Dẫn
The Following 3 Users Say Thank You to Short_list For This Useful Post:
namdung (27-10-2014), thaibinh (15-10-2014), thaygiaocht (14-10-2014)
Old 03-11-2014, 11:12 PM   #11
nguyentatthu
Super Moderator
 
Tham gia ngày: Nov 2007
Đến từ: BH
Bài gởi: 212
Thanks: 135
Thanked 345 Times in 92 Posts
Trích:
Nguyên văn bởi namdung View Post
Tiếp tục một số bài toán chọn lọc


22. (Gia Lai) Xét tất cả các tập hợp $ X = {a_1, a_2, ..., a_n} $ các số nguyên dương có tính chất: khi ta bỏ đi một phần tử bất kỳ trong X thì tập còn lại có thể phân hoạch thành 2 tập con khác rỗng sao cho tổng các phần tử trong mỗi tập con đó bằng nhau. Hãy tìm giá trị nhỏ nhất của n.
Ta có phần tử ${{a}_{i}}$ lẻ (vì nếu các ${{a}_{i}}$ đều chẵn ta xét tập $\left\{ \frac{{{a}_{i}}}{{{2}^{k}}} \right\}$).
Suy ra $S\left( X \right)-{{a}_{i}}\vdots 2\Rightarrow S(X)$ lẻ.
Mà với mọi $j\ne i$ thì $S(X)-{{a}_{j}}\vdots 2\Rightarrow {{a}_{j}}$ lẻ.
Suy ra $\sum\limits_{k=1}^{n}{\left( S(X)-{{a}_{k}} \right)}=\left( n-1 \right)S(X)\vdots 2\Rightarrow n$ lẻ, dẫn đến $n\ge 5$.
+) $n=5$ ta có $X=\left\{ {{a}_{1}}<{{a}_{2}}<{{a}_{3}}<{{a}_{4}}<{{a}_{5}} \right\}$.
Bỏ ${{a}_{1}}$ thì ta có ${{a}_{2}}+{{a}_{5}}={{a}_{3}}+{{a}_{4}}$
Bỏ ${{a}_{2}}$ ta có ${{a}_{1}}+{{a}_{5}}={{a}_{3}}+{{a}_{4}}\Rightarro w {{a}_{1}}={{a}_{2}}$ (vô lí).
+) $n=7$ ta xét tập $X=\left\{ 1,3,5,7,9,11,13 \right\}$. Ta thấy $X$ thỏa mãn bài toán.
[RIGHT][I][B]Nguồn: MathScope.ORG[/B][/I][/RIGHT]
 
nguyentatthu is offline   Trả Lời Với Trích Dẫn
The Following User Says Thank You to nguyentatthu For This Useful Post:
namdung (11-11-2014)
Old 19-10-2014, 07:44 AM   #12
Livetolove2207
+Thành Viên+
 
Livetolove2207's Avatar
 
Tham gia ngày: Jul 2014
Đến từ: THPT chuyên Lê Quý Đôn tỉnh Ninh Thuận
Bài gởi: 63
Thanks: 65
Thanked 12 Times in 9 Posts
Em xin đóng góp một bài toán sau:

Cho $x, y, z$ là các số không âm. Chứng minh rằng:
$xyz+x^{2}+y^{2}+z^{2}+5\geq 3\left ( x+y+z \right )$

(Đề chọn Đội tuyển trường THPT chuyên Lê Quý Đôn tỉnh Ninh Thuận)
[RIGHT][I][B]Nguồn: MathScope.ORG[/B][/I][/RIGHT]
 
__________________
Có Đức mà không có Tài, làm việc gì cũng khó;
Có Tài mà không có Đức, là vô dụng. (Hồ Chí Minh)
Livetolove2207 is offline   Trả Lời Với Trích Dẫn
The Following User Says Thank You to Livetolove2207 For This Useful Post:
Thmcuongvn (19-10-2014)
Old 19-10-2014, 02:50 PM   #13
Juliel
+Thành Viên+
 
Tham gia ngày: Sep 2013
Đến từ: THPT Chuyên Lương Thế Vinh, Biên Hoà, Đồng Nai
Bài gởi: 144
Thanks: 109
Thanked 130 Times in 66 Posts
Trích:
Nguyên văn bởi Livetolove2207 View Post
Em xin đóng góp một bài toán sau:

Cho $x, y, z$ là các số không âm. Chứng minh rằng:
$xyz+x^{2}+y^{2}+z^{2}+5\geq 3\left ( x+y+z \right )$

(Đề chọn Đội tuyển trường THPT chuyên Lê Quý Đôn tỉnh Ninh Thuận)
Bất đẳng thức sau có lẽ quen thuộc :
$$x^2+y^2+z^2+2xyz+1\geq 2(xy+yz+zx)$$
Từ đây suy ra :
$$xyz\geq \dfrac{2(xy+yz+zx)-1-x^2-y^2-z^2}{2}$$
vậy ta đi chứng minh :

$$\dfrac{2(xy+yz+zx)-1-x^2-y^2-z^2}{2}+x^2+y^2+z^2+5\geq 6(x+y+z)$$
$$\Leftrightarrow (x+y+z)^2+9\geq 6(x+y+z)\Leftrightarrow (x+y+z-3)^2\geq 0$$
Luôn đúng. Điều phải chứng minh.
[RIGHT][I][B]Nguồn: MathScope.ORG[/B][/I][/RIGHT]
 
Juliel is offline   Trả Lời Với Trích Dẫn
The Following 2 Users Say Thank You to Juliel For This Useful Post:
Livetolove2207 (19-10-2014), thaygiaocht (19-10-2014)
Old 26-10-2014, 09:53 PM   #14
namdung
Administrator

 
Tham gia ngày: Feb 2009
Đến từ: Tp Hồ Chí Minh
Bài gởi: 1,343
Thanks: 209
Thanked 4,066 Times in 778 Posts
Gửi tin nhắn qua Yahoo chát tới namdung
Bài tổ hợp PTNK có cách giải khá hay.

Đặt F(k) là tập các tập con A có 7 phần tử của X sao cho tổng các phần tử của A đồng dư k mod 19.

Khi đó các tập thuộc F(k) sẽ thỏa mãn điều kiện đề bài (tại sao?)

Nhưng $ |F(0)| + |F(1)| + ... + |F(18)| = C^7_19 $

Từ đó tồn tại k sao cho $ |F(k)| \le \frac{C^7_{19}}{19} = 2652 $.

Một số câu hỏi thú vị được đặt ra.
1) |F(k)| = ? với k = 0, 1, ..., 18.
2) Có thể nâng số 2652 lên không? Và chặn trên của số này là bao nhiêu?
[RIGHT][I][B]Nguồn: MathScope.ORG[/B][/I][/RIGHT]
 
namdung is offline   Trả Lời Với Trích Dẫn
Old 26-10-2014, 10:42 PM   #15
vantienducdh
+Thành Viên+
 
Tham gia ngày: Aug 2014
Đến từ: 12 Toán THPT chuyên LQĐ-Quảng Trị
Bài gởi: 45
Thanks: 35
Thanked 11 Times in 10 Posts
Trích:
Nguyên văn bởi namdung View Post
Hôm nay là ngày 1/10, ngày tốt để bắt đầu một công việc mà chúng ta đã làm đều đặn nhiều năm qua: Giải và bình luận đề thi các trường và các tỉnh.

Cuối tháng 9 vừa qua đã có nhiều trường và nhiều tỉnh thi chọn đội tuyển như Đà Nẵng, Bắc Giang, Quảng Trị, Ninh Bình, Hà Tĩnh, Đà Lạt, Đồng Nai, Sóc Trăng, PTNK ... và sắp tới hàng loạt các trường, các tỉnh sẽ thi. Năm nay chúng ta sẽ đổi mới một chút trong việc giải và bình luận. Ta sẽ coi trọng hơn về chất lượng chứ không chạy đua theo số lượng. Vì thế, ta sẽ chọn lọc những bài hay, có ý nghĩa học thuật nhiều hơn để bình luận và thật chi tiết, với cả những vấn đề xung quang, những sai lầm thường gặp. Như vậy các bạn học sinh và các thầy cô giáo sẽ thu nhận được nhiều hơn là đơn thuần một lời giải.

Tôi xin chọn lọc trước ở đây một số bài để các bạn vào giải và bình luận. Các bạn cũng có thể đề xuất các bài toán khác cần có bình luận. Để chủ đề không bị loãng, mong các bạn chọn lọc kỹ một chút. Chú ý là ta không quan tâm nhiều đến số lượng. Quan điểm của chúng ta là không chỉ chọn những bài khó và mới, mà là những bài qua đó có thể giảng giải cho các bạn học sinh được nhiều nhất.

1. (Đồng Tháp) Cho x, y, z là các số thực thoả mãn điều kiện $x + y + z = 5, x^2 + y^2 + z^2 = 9$ . Tìm giá trị lớn nhất và giá trị nhỏ nhất của biểu thức $ P = x^3+ y^3 + z^3 $.

2. (Đồng Tháp) Có một số số thực dương, mỗi số không lớn hơn 0,5 và tổng của chúng không vượt quá 2,5.
a) Chứng minh rằng ta có thể chia các số thành 4 nhóm, mỗi nhóm có tổng các số không vượt quá 1.
b) Khẳng định trên còn đúng không nếu thay 4 nhóm bằng 3 nhóm?

3. (Quảng Trị) Chứng minh bất đẳng thức sau $3(x^2-x+1)(y^2-y+1) \ge 2(x^2y^2-xy+1)$ với mọi x, y thực.

4. (Quảng Trị) Ban đầu trên bảng điện tử hiển thị hai số phân biệt a, b. Sau mỗi giây, bảng sẽ tự động hiển thị thêm các số n nếu n chưa có trên bảng và là tổng của hai số đã có trên bảng. Hãy xét xem số 2014 có được hiển thị trên bảng không, nếu có thì sau thời gian ít nhất bao lâu (kể từ thời điểm ban đầu), trong các trường hợp sau a) a = 3, b = 12; b) a = 1, b = 2.

5. (PTNK) Cho a, b, c là các số thực dương thỏa mãn điều kiện $ (a+1)(b+1)(c+1) = 1 + 4abc $ Chứng minh rằng ta có bất đẳng thức $ a + b + c \le 1 + abc $

6. (PTNK) Tìm tất cả các hàm số f: N* --> N* thỏa mãn hệ thức $ f(f(n)/n) = n^2 $ với mọi n nguyên dương. N* ký hiệu tập hợp các số nguyên dương.

7. (Sóc Trăng) Sau khi thi tốt nghiệp trung hoc phổ thông, các em học sinh khối 12 của trường trung học phổ thông MTT hẹn nhau mỗi năm họp mặt 1 lần vào ngày hội trại của trường. Tuy nhiên các thành viên không có mặt đủ theo giao ước ban đầu mà chỉ có đúng 20 thành viên tham dự mỗi lần họp mặt. Qua 10 lần họp mặt không có 2 thành viên nào dự họp mặt với nhau quá một lần. Hỏi khối 12 năm học đó của trường trung học phổ thông MTT có ít nhất là bao nhiêu học sinh?

8. (Đà Nẵng) Tại một hội nghị quốc tế, các đại biểu tham dự biết ít nhất một trong 3 thứ tiếng Anh, Pháp, Đức. Biết rằng có đúng 50 đại biểu biết tiếng Anh, đúng 50 đại biểu biết tiếng Pháp và đúng 50 đại biểu biết tiếng Đức. Chứng minh rằng có thể chia các đại biểu thành 5 nhóm sao cho trong mỗi nhóm có đúng 10 đại biểu biết tiếng Anh, đúng 10 đại biểu biết tiếng Pháp và đúng 10 đại biểu biết tiếng Đức.

9. (Đà Nẵng) Các ô vuông của bảng vuông kích thước 10 x 10 được tô bằng các màu trắng và đen sao cho trên mỗi hàng cũng như trên mỗi cột đều có đúng ba ô được tô màu đen. Chứng minh rằng trong mọi cách tô như vậy luôn có thể chọn ra 10 ô được tô màu đen sao cho không có hai ô nào được chọn cùng hàng hoặc cùng cột.

10. (Đội tuyển Lương Thế Vinh, Đồng Nai) Tìm tất cả các hàm số f: R --> R thỏa mãn điều kiện f(f(x) - y) + f(x+y) = 2x với mọi x, y thuộc R.

11. (???) Cho dãy số $ u_n $ được xác định bởi $ u_1 = \frac{21}{10} $, $ u_{n+1} = \frac{u_n-2+\sqrt{u_n^2+8u_n-4}}{2}$ với mọi n = 1, 2, 3, ... Đặt $v_n = \frac{1}{u_2^2-4} + ... + \frac{1}{u_{n+1}^2-4} $ Tìm $ lim v_n $ khi n dần đến vô cùng.

12. (Đà Lạt) Cho a, b, c là các số thực dương thỏa mãn điều kiện a + b + c = 3. Chứng minh rằng ta có bất đẳng thức $a^3 + b^3 + c^3 + 2(\frac{1}{a} + \frac{1}{b} + \frac{1}{c}) \ge 3(ab+bc+ca)$

13. (Đà Lạt) Từ các chữ số 1, 3, 5, 9 có thể lập được bao nhiêu số tự nhiên chia hết cho 3, mỗi số có 2014 chữ số.

Rất mong sự hưởng ứng của các thầy cô, các bạn sinh viên, các cựu IMO, cựu VMO và các bạn học sinh.
bài tổ hợp số 7 của Sóc Trăng hình như là 1 dạng như trong bài tổ hợp của USAMO 2001,nhưng có vẻ thiếu thì phải
[RIGHT][I][B]Nguồn: MathScope.ORG[/B][/I][/RIGHT]
 
vantienducdh is offline   Trả Lời Với Trích Dẫn
Trả lời Gởi Ðề Tài Mới

Bookmarks


Quuyền Hạn Của Bạn
You may not post new threads
You may not post replies
You may not post attachments
You may not edit your posts

BB code is Mở
Smilies đang Mở
[IMG] đang Mở
HTML đang Tắt

Chuyển đến


Múi giờ GMT. Hiện tại là 01:41 AM.


Powered by: vBulletin Copyright ©2000-2024, Jelsoft Enterprises Ltd.
Inactive Reminders By mathscope.org
[page compression: 129.37 k/146.40 k (11.63%)]